1answer.
Ask question
Login Signup
Ask question
All categories
  • English
  • Mathematics
  • Social Studies
  • Business
  • History
  • Health
  • Geography
  • Biology
  • Physics
  • Chemistry
  • Computers and Technology
  • Arts
  • World Languages
  • Spanish
  • French
  • German
  • Advanced Placement (AP)
  • SAT
  • Medicine
  • Law
  • Engineering
Nostrana [21]
3 years ago
8

X-3/4 = 2x-9/7 Please answer it I’ll mark your answer as BRAINLIEST

Mathematics
2 answers:
lidiya [134]3 years ago
7 0
<h3><u>Answer</u> :</h3>

:\implies\sf\:x-\dfrac{3}{4}=2x-\dfrac{9}{7}

:\implies\sf\:\dfrac{9}{7}-\dfrac{3}{4}=2x-x

:\implies\sf\:\dfrac{36-21}{28}=x

:\implies\boxed{\bf{\red{x=\dfrac{15}{28}}}}

cluponka [151]3 years ago
7 0
Combine like terms

-3/4+9/7=x

add fractions

15/28=x
You might be interested in
How do you solve this?
Sholpan [36]

40% of 60 is 24


To convert from percent to decimal, divide the percent without the percent symbol by  100.

so, 40% = 40/100 = 0.4

40% 60 is just 0.4*60 = 24

6 0
3 years ago
Read 2 more answers
A high school athletic department bought 40 soccer uniforms at a cost of $3,000. After soccer season they returned some uniforms
quester [9]

Interesting problem.

First - let's figure cost of each uniform at purchase.

3,000/40 = $75 each

When some uniforms were returned at $40 - there was a difference of $35 in what they paid and what they rec'd in return. ($75 - 35 = $40)

8 0
3 years ago
(a) How many integers in the range 1 through 120 are integer multiples of 2, 3, or 5? keyboard_arrow_down Solution (b) How many
Umnica [9.8K]

Answer:

(a) 88 integers

(b) 92 integers

Step-by-step explanation:

(a) integers whose last digits are divisible by 2 are multiples of two or numbers whose digits ends with zero. So for number 1-120 , all the even numbers which are sixty in number are are multiples of two.

For  3, numbers whose digits sum is divisible by three are multiples of three. 3,6,9,12,15,18,21,24,27,30 are multiples of three from numbers 1-30. we have four 30s in 120. which means numbers of integers will be 10*4 = 40integers. However out of these numbers , half are also integers of 2 which reduces the number added to 20integers.

For 5, numbers whose digits ends with 5 or 0 are multiples of 5. this gives us 24 integers for 1-120. but out of these 24integers, 16 are common integers of 2 and 3 which reduces the number added to 8integers.

Thus from 1-120 the intergers of 2,3 or 5 = 60+20+8 = 88integers.

(b) if we are considering from numbers 1-140;

for 2 we wil have 70 integers,

for 5 we will have 28 integers, but those integers that end with 0 are also integers of 2 which reduces the number added to 14.

For 7, numbers 7,14,21,28,35,42,49,56,63,70 are multiples of three from 1-70. This pattern is repeated from number 71-140. hence we have 20 integers in all. However 12 of the multiples are also multiples of either 2 or 5 which reduces the number to 8 integers.

Thus from 1-140, the integers of 2, 5, or 7 = 70+14+8 = 92integers

3 0
3 years ago
How to solve for -3x squared + 2y squared + 5xy - 2y + 5x squared - 3y squared when x = 0.5 and y = -1/10??
Pani-rosa [81]

Answer:

0.44

Step-by-step explanation:

-3x^2 + 2y^2 + 5xy - 2y +5x^2 - 3y^2

Combine like terms

-3x^2 + 5x^2 = 2x^2 2y^2 - 3y^2 = -1y^2

2x^2 - 1y^2 + 5xy - 2y

Now plug in the solutions Note: it is easier if you have all decimals or all fractions (-1/10=-.1

2(0.5)^2 - 1(-0.1)^2 + 5(0.5)(-0.1) - 2(-0.1)

Simplify:

0.5 - 0.01 - 0.25 + 0.2

0.5 + 0.2 - 0.01 - 0.25

0.7 - 0.26

0.44

5 0
3 years ago
GEO, TRIGONOMETRY! 45 POINTS AND BRAINLIEST FOR CORRECT ANSWER. NEED HELP ASAP PLEASE :) Isabell is standing at the top of a wat
IRINA_888 [86]

Answer:

h ≈ 39 ft

Step-by-step explanation:

When we draw out a picture, we should see that we have to use cos∅ to solve this:

cos31.66° = x/46

46cos31.66° = x

x = 39.1542

5 0
3 years ago
Other questions:
  • Two coins are flipped. What is the probability that both coins land heads up? *<br> 1 point
    5·2 answers
  • The school carnival was almost closing and they had 3 pounds of candy remaining they give it all away but letting the 14 kids at
    14·1 answer
  • the city of James town is 2 meters below sea level. Takoradi, a city in western region, is 7 meters below sea level . How much h
    15·1 answer
  • Delaney has 4 times as many books as she has movies. If she buys 15 more movies ( but no more books ), her ratio of movies to bo
    14·1 answer
  • Using distance formula i tried this one for myself and just couldn't get it
    12·1 answer
  • Timo was curious if quadrilaterals LMNR and PQNO were similar, so he tried to map one figure onto the
    9·2 answers
  • Help me with geometry this is my 3rd time asking
    6·1 answer
  • A transformation that does not change the size or shape of a geometric object is known as which of the following?
    14·1 answer
  • 3. What is the greatest common factor of 15 and 30? *
    9·2 answers
  • Can someone help me solve this please
    8·1 answer
Add answer
Login
Not registered? Fast signup
Signup
Login Signup
Ask question!